2006 AMC 8 Problems/Problem 1

Revision as of 13:14, 18 March 2020 by Emerald block (talk | contribs) (fixed small grammar mistake and added clarification)
(diff) ← Older revision | Latest revision (diff) | Newer revision → (diff)

Problem

Mindy made three purchases for $\textdollar 1.98$ dollars, $\textdollar 5.04$ dollars, and $\textdollar 9.89$ dollars. What was her total, to the nearest dollar?

$\textbf{(A)}\ 10\qquad\textbf{(B)}\ 15\qquad\textbf{(C)}\ 16\qquad\textbf{(D)}\ 17\qquad\textbf{(E)}\ 18$

Solution

The three prices round to $\textdollar 2$, $\textdollar 5$, and $\textdollar 10$, which have a sum of $\boxed{\textbf{(D)}\ 17}$.

We know that there will not be a rounding error, as the total amount rounded is clearly less than $\textdollar 0.50$.

See Also

2006 AMC 8 (ProblemsAnswer KeyResources)
Preceded by
First
Question
Followed by
Problem 2
1 2 3 4 5 6 7 8 9 10 11 12 13 14 15 16 17 18 19 20 21 22 23 24 25
All AJHSME/AMC 8 Problems and Solutions

The problems on this page are copyrighted by the Mathematical Association of America's American Mathematics Competitions. AMC logo.png